Find $P(X=Y+1)$











up vote
1
down vote

favorite












Suppose that X and Y are integer valued random variables with joint probability mass function given by
$p_{X,Y}(a, b)=begin{cases} frac{1}{4a}, & 1leq bleq aleq 4\ 0, & text{otherwise}. end{cases}.$



(b) Find the marginal probability mass functions of X and Y.
(c) Find $P(X=Y+1)$



Since this is a discrete random variable, I want to construct a table of joint distribution to find the sum of rows, columns for pmf of X and Y, but I'm having trouble with it. Anyone can help with part (b) and (c)?










share|cite|improve this question


























    up vote
    1
    down vote

    favorite












    Suppose that X and Y are integer valued random variables with joint probability mass function given by
    $p_{X,Y}(a, b)=begin{cases} frac{1}{4a}, & 1leq bleq aleq 4\ 0, & text{otherwise}. end{cases}.$



    (b) Find the marginal probability mass functions of X and Y.
    (c) Find $P(X=Y+1)$



    Since this is a discrete random variable, I want to construct a table of joint distribution to find the sum of rows, columns for pmf of X and Y, but I'm having trouble with it. Anyone can help with part (b) and (c)?










    share|cite|improve this question
























      up vote
      1
      down vote

      favorite









      up vote
      1
      down vote

      favorite











      Suppose that X and Y are integer valued random variables with joint probability mass function given by
      $p_{X,Y}(a, b)=begin{cases} frac{1}{4a}, & 1leq bleq aleq 4\ 0, & text{otherwise}. end{cases}.$



      (b) Find the marginal probability mass functions of X and Y.
      (c) Find $P(X=Y+1)$



      Since this is a discrete random variable, I want to construct a table of joint distribution to find the sum of rows, columns for pmf of X and Y, but I'm having trouble with it. Anyone can help with part (b) and (c)?










      share|cite|improve this question













      Suppose that X and Y are integer valued random variables with joint probability mass function given by
      $p_{X,Y}(a, b)=begin{cases} frac{1}{4a}, & 1leq bleq aleq 4\ 0, & text{otherwise}. end{cases}.$



      (b) Find the marginal probability mass functions of X and Y.
      (c) Find $P(X=Y+1)$



      Since this is a discrete random variable, I want to construct a table of joint distribution to find the sum of rows, columns for pmf of X and Y, but I'm having trouble with it. Anyone can help with part (b) and (c)?







      probability






      share|cite|improve this question













      share|cite|improve this question











      share|cite|improve this question




      share|cite|improve this question










      asked Nov 14 at 4:18









      dxdydz

      989




      989






















          1 Answer
          1






          active

          oldest

          votes

















          up vote
          1
          down vote



          accepted










          The table is not hard.   Each cell in 4 rows ($1leq aleq 4$) of $a$ columns ($1leq bleq a$), contains $tfrac 1{4a}$, the rest of the cells are zero .



          $$p_{X,Y}(a,b)=tfrac 1{4a}mathbf 1_{1leq bleq aleq 4}\boxed{begin{array}{c|c:c:c:c|c}abackslash b & 1 & 2 & 3 & 4 &\hline 1 & tfrac 14 &0&0&0\ hdashline 2 & tfrac 18&tfrac 18&0&0\ hdashline 3 &tfrac 1{12}&tfrac 1{12}&tfrac 1{12}&0\ hdashline 4 & tfrac 1{16}&tfrac 1{16}&tfrac 1{16}&tfrac 1{16}\hline ~&&&&&1end{array}}$$






          share|cite|improve this answer



















          • 1




            So $P(X=Y+1) = P(X=2, Y=1) + P(X=3, Y=2) + P(X=4, Y=3) = 1/8 + 1/12 +1/16 = 13/48$?
            – dxdydz
            Nov 14 at 5:08










          • Indeed. @dxdydz
            – Graham Kemp
            Nov 14 at 11:59











          Your Answer





          StackExchange.ifUsing("editor", function () {
          return StackExchange.using("mathjaxEditing", function () {
          StackExchange.MarkdownEditor.creationCallbacks.add(function (editor, postfix) {
          StackExchange.mathjaxEditing.prepareWmdForMathJax(editor, postfix, [["$", "$"], ["\\(","\\)"]]);
          });
          });
          }, "mathjax-editing");

          StackExchange.ready(function() {
          var channelOptions = {
          tags: "".split(" "),
          id: "69"
          };
          initTagRenderer("".split(" "), "".split(" "), channelOptions);

          StackExchange.using("externalEditor", function() {
          // Have to fire editor after snippets, if snippets enabled
          if (StackExchange.settings.snippets.snippetsEnabled) {
          StackExchange.using("snippets", function() {
          createEditor();
          });
          }
          else {
          createEditor();
          }
          });

          function createEditor() {
          StackExchange.prepareEditor({
          heartbeatType: 'answer',
          convertImagesToLinks: true,
          noModals: true,
          showLowRepImageUploadWarning: true,
          reputationToPostImages: 10,
          bindNavPrevention: true,
          postfix: "",
          imageUploader: {
          brandingHtml: "Powered by u003ca class="icon-imgur-white" href="https://imgur.com/"u003eu003c/au003e",
          contentPolicyHtml: "User contributions licensed under u003ca href="https://creativecommons.org/licenses/by-sa/3.0/"u003ecc by-sa 3.0 with attribution requiredu003c/au003e u003ca href="https://stackoverflow.com/legal/content-policy"u003e(content policy)u003c/au003e",
          allowUrls: true
          },
          noCode: true, onDemand: true,
          discardSelector: ".discard-answer"
          ,immediatelyShowMarkdownHelp:true
          });


          }
          });














           

          draft saved


          draft discarded


















          StackExchange.ready(
          function () {
          StackExchange.openid.initPostLogin('.new-post-login', 'https%3a%2f%2fmath.stackexchange.com%2fquestions%2f2997775%2ffind-px-y1%23new-answer', 'question_page');
          }
          );

          Post as a guest















          Required, but never shown

























          1 Answer
          1






          active

          oldest

          votes








          1 Answer
          1






          active

          oldest

          votes









          active

          oldest

          votes






          active

          oldest

          votes








          up vote
          1
          down vote



          accepted










          The table is not hard.   Each cell in 4 rows ($1leq aleq 4$) of $a$ columns ($1leq bleq a$), contains $tfrac 1{4a}$, the rest of the cells are zero .



          $$p_{X,Y}(a,b)=tfrac 1{4a}mathbf 1_{1leq bleq aleq 4}\boxed{begin{array}{c|c:c:c:c|c}abackslash b & 1 & 2 & 3 & 4 &\hline 1 & tfrac 14 &0&0&0\ hdashline 2 & tfrac 18&tfrac 18&0&0\ hdashline 3 &tfrac 1{12}&tfrac 1{12}&tfrac 1{12}&0\ hdashline 4 & tfrac 1{16}&tfrac 1{16}&tfrac 1{16}&tfrac 1{16}\hline ~&&&&&1end{array}}$$






          share|cite|improve this answer



















          • 1




            So $P(X=Y+1) = P(X=2, Y=1) + P(X=3, Y=2) + P(X=4, Y=3) = 1/8 + 1/12 +1/16 = 13/48$?
            – dxdydz
            Nov 14 at 5:08










          • Indeed. @dxdydz
            – Graham Kemp
            Nov 14 at 11:59















          up vote
          1
          down vote



          accepted










          The table is not hard.   Each cell in 4 rows ($1leq aleq 4$) of $a$ columns ($1leq bleq a$), contains $tfrac 1{4a}$, the rest of the cells are zero .



          $$p_{X,Y}(a,b)=tfrac 1{4a}mathbf 1_{1leq bleq aleq 4}\boxed{begin{array}{c|c:c:c:c|c}abackslash b & 1 & 2 & 3 & 4 &\hline 1 & tfrac 14 &0&0&0\ hdashline 2 & tfrac 18&tfrac 18&0&0\ hdashline 3 &tfrac 1{12}&tfrac 1{12}&tfrac 1{12}&0\ hdashline 4 & tfrac 1{16}&tfrac 1{16}&tfrac 1{16}&tfrac 1{16}\hline ~&&&&&1end{array}}$$






          share|cite|improve this answer



















          • 1




            So $P(X=Y+1) = P(X=2, Y=1) + P(X=3, Y=2) + P(X=4, Y=3) = 1/8 + 1/12 +1/16 = 13/48$?
            – dxdydz
            Nov 14 at 5:08










          • Indeed. @dxdydz
            – Graham Kemp
            Nov 14 at 11:59













          up vote
          1
          down vote



          accepted







          up vote
          1
          down vote



          accepted






          The table is not hard.   Each cell in 4 rows ($1leq aleq 4$) of $a$ columns ($1leq bleq a$), contains $tfrac 1{4a}$, the rest of the cells are zero .



          $$p_{X,Y}(a,b)=tfrac 1{4a}mathbf 1_{1leq bleq aleq 4}\boxed{begin{array}{c|c:c:c:c|c}abackslash b & 1 & 2 & 3 & 4 &\hline 1 & tfrac 14 &0&0&0\ hdashline 2 & tfrac 18&tfrac 18&0&0\ hdashline 3 &tfrac 1{12}&tfrac 1{12}&tfrac 1{12}&0\ hdashline 4 & tfrac 1{16}&tfrac 1{16}&tfrac 1{16}&tfrac 1{16}\hline ~&&&&&1end{array}}$$






          share|cite|improve this answer














          The table is not hard.   Each cell in 4 rows ($1leq aleq 4$) of $a$ columns ($1leq bleq a$), contains $tfrac 1{4a}$, the rest of the cells are zero .



          $$p_{X,Y}(a,b)=tfrac 1{4a}mathbf 1_{1leq bleq aleq 4}\boxed{begin{array}{c|c:c:c:c|c}abackslash b & 1 & 2 & 3 & 4 &\hline 1 & tfrac 14 &0&0&0\ hdashline 2 & tfrac 18&tfrac 18&0&0\ hdashline 3 &tfrac 1{12}&tfrac 1{12}&tfrac 1{12}&0\ hdashline 4 & tfrac 1{16}&tfrac 1{16}&tfrac 1{16}&tfrac 1{16}\hline ~&&&&&1end{array}}$$







          share|cite|improve this answer














          share|cite|improve this answer



          share|cite|improve this answer








          edited Nov 14 at 5:06

























          answered Nov 14 at 5:00









          Graham Kemp

          84.2k43378




          84.2k43378








          • 1




            So $P(X=Y+1) = P(X=2, Y=1) + P(X=3, Y=2) + P(X=4, Y=3) = 1/8 + 1/12 +1/16 = 13/48$?
            – dxdydz
            Nov 14 at 5:08










          • Indeed. @dxdydz
            – Graham Kemp
            Nov 14 at 11:59














          • 1




            So $P(X=Y+1) = P(X=2, Y=1) + P(X=3, Y=2) + P(X=4, Y=3) = 1/8 + 1/12 +1/16 = 13/48$?
            – dxdydz
            Nov 14 at 5:08










          • Indeed. @dxdydz
            – Graham Kemp
            Nov 14 at 11:59








          1




          1




          So $P(X=Y+1) = P(X=2, Y=1) + P(X=3, Y=2) + P(X=4, Y=3) = 1/8 + 1/12 +1/16 = 13/48$?
          – dxdydz
          Nov 14 at 5:08




          So $P(X=Y+1) = P(X=2, Y=1) + P(X=3, Y=2) + P(X=4, Y=3) = 1/8 + 1/12 +1/16 = 13/48$?
          – dxdydz
          Nov 14 at 5:08












          Indeed. @dxdydz
          – Graham Kemp
          Nov 14 at 11:59




          Indeed. @dxdydz
          – Graham Kemp
          Nov 14 at 11:59


















           

          draft saved


          draft discarded



















































           


          draft saved


          draft discarded














          StackExchange.ready(
          function () {
          StackExchange.openid.initPostLogin('.new-post-login', 'https%3a%2f%2fmath.stackexchange.com%2fquestions%2f2997775%2ffind-px-y1%23new-answer', 'question_page');
          }
          );

          Post as a guest















          Required, but never shown





















































          Required, but never shown














          Required, but never shown












          Required, but never shown







          Required, but never shown

































          Required, but never shown














          Required, but never shown












          Required, but never shown







          Required, but never shown







          Popular posts from this blog

          Biblatex bibliography style without URLs when DOI exists (in Overleaf with Zotero bibliography)

          ComboBox Display Member on multiple fields

          Is it possible to collect Nectar points via Trainline?